A and B contributed 2700 and 1800 respectively to start a business. They agreed to share the profit in the ratio 7:5 respectively. If the profit made was 900, Calculate B's share of the profit.​

Answers

Answer 1

Answer:

375

Step-by-step explanation:

let the ratio constant be 'x'

=>Profit ratio for A=7x

=>Profit ratio for B=5x

Profit=900 (Given)

Sum of the ratios=7x+5x=12x

Therefore,

Profit ratio for A=(7/12)x900=7x75=525

Profit ratio for B=(5/12)x900=7x75=375


Related Questions

1, 3, 2, 2, 1, 3, 3, 1

Part A: What is the mean of the data? Show your work. (4 points)

Part B: Use your answer from Part A to calculate the mean absolute deviation for the data. Show your work. (6 points)

Answers

Answer:

(I only know part A. sorry) The mean is 2.

Step-by-step explanation:

To find the mean of data you have to add all of the numbers and then divide by the total number of numbers being added. (I hope that made sense)

So you have to do- 1+3+2+2+1+3+3+1=16 -and then- 16÷8=2 -so you're answer is 2.

Suppose y varies directly with x, and y = 45 when x = 3. What is the value of y when x = -6?

Answers

Answer:

y=-90

Step-by-step explanation:

y = kx

45 = k 3

k = 15

~~~~~~~~~~~~

y = 15*-6

y=-90

Answer:

36

We should write a formula for their relationship:

Y = x+42. <- this is what a direct varieation is I believe.

So, when x is -6, y is +42 aka 36

The potential solutions to the radical equation are a = −4 and a = −1. Which statement is true about these solutions? The solution a = −4 is an extraneous solution. The solution a = −1 is an extraneous solution. Both a = −4 and a = −1 are true solutions. Neither a = −4 nor a = −1 are true solutions.

Answers

Answer:

Both a = −4 and a = −1 are true solutions

Step-by-step explanation:

Given

[tex]\sqrt{a + 5} = a + 3[/tex]

[tex]a = -4; a = -1[/tex]

Required

The true statement about the solutions

We have:

[tex]\sqrt{a + 5} = a + 3[/tex]

Square both sides

[tex]a + 5 = (a + 3)^2[/tex]

[tex]a + 5 =a^2 + 6a + 9[/tex]

Collect like terms

[tex]a^2 + 6a - a + 9 - 5 = 0[/tex]

[tex]a^2 + 5a + 4 = 0[/tex]

Expand

[tex]a^2 + 4a + a + 4 = 0[/tex]

Factorize

[tex]a(a + 4) + 1(a + 4) = 0[/tex]

Factor out a + 4

[tex](a + 1)(a + 4) = 0[/tex]

Split:

[tex]a + 1 = 0; a + 4 = 0[/tex]

Solve:

[tex]a =-1; a = -4[/tex]

Answer:

C on 3dge

Step-by-step explanation:

Both a = −4 and a = −1 are true solutions

Pleaseeeeee help meee

Answers

Answer:

Surface area of sphere = 1,344.8 m² (Approx)

Step-by-step explanation:

Given:

Volume of sphere = 1,476π m³

Find;

Surface area of sphere

Computation:

Volume of sphere = [4/3]πr³

1,476π = [4/3]πr³

1,476 = [4/3]r³

[1,476][3/4] = r³

1,107 = r³

r = 10.35 m (Approx.)

Surface area of sphere = 4πr²

Surface area of sphere = 4(3.14)(10.35)²

Surface area of sphere = 4(3.14)(107.1225)

Surface area of sphere = 1,345.45

Surface area of sphere = 1,344.8 m² (Approx)

Helpppppp pls
Find h.
h = [?] in

Answers

Answer:

h=rt.96=4 rt.6

Step-by-step explanation:

r=2 inches

therefore 100= root over h + 4

          =>h=rt.96=4 rt.6

Please help I’ll give branliest

Answers

Answer:

optin d) 8

Step-by-step explanation:

           [tex]2 \times 2 \times 2 = 2^3[/tex] [tex]= 8[/tex]

Answer: 8

Explanation:
2x2= 4
4x2= 8
Split up the equation to help:) Since its regular multiplication it doesnt effect the product by doing it in steps

Simplify -|-7 + 4|. please helppppp

Answers

Answer: -3

Step-by-step explanation:

-l-7+4l = -l-3l = - 3

ll means anything inside there is positive, so for example if -3 was in there, l-3l = 3

Answer:

- 3

Step-by-step explanation:

-|-7 + 4|

Simplify inside the absolute values first( like parentheses)

-|-3|

Take the nonnegative value for the absolute value

- 3

How do you solve this?

Answers

Answer:73825

Step-by-step explanation:

Shuts shield grid

Answer:

[tex]j=8[/tex]

Step-by-step explanation:

Special right triangles:

The sides of all 30-60-90 triangles are in the proportion [tex]x:x\sqrt{3}:2x[/tex], where [tex]2x[/tex] is the hypotenuse of the triangle and [tex]x[/tex] is the side directly opposite to the 30 degree angle.

Since the base of the triangle is directly opposite to the 30 degree angle, the hypotenuse, j, must be equal to [tex]4\cdot 2=\boxed{8}[/tex]

Basic trig:

[tex]\sin 30^{\circ}=\frac{4}{j},\\j=\frac{4}{\sin 30^{\circ}}=\boxed{8}[/tex]

Help please !!!!!!!!!!!!

Answers

Answer:  8.50

=============================================================

Explanation:

Ignore the rows that are labeled "stock ABC" and "stock MNO".

We only focus on the top row (gains and losses) and the "stock JKL" row.

Multiply each gain or loss with its corresponding probability in decimal form. Use negative values to represent a loss

-25*0.15 = -3.755*0.65 = 3.2545*0.20 = 9

Add up those results to get the final answer: -3.75+3.25+9 = 8.50

-----------------------

Here's another way to get the answer.

Let's say you had a 10 by 10 grid of graph paper.

Shade exactly 15 tiny squares in red to represent a loss of $25 (note that 15/100 = 15%)Shade exactly 65 squares in blue to represent a gain of $5  (65/100 = 65%)Shade the remaining 20 squares in green to represent a gain of $45

This can be thought of as like a dart board. Imagine throwing 1000 darts at this grid (maybe do so blindfolded to give each square an equal chance; if you land outside the grid then re-throw the dart). Theoretically, the probability of landing in the red area is 15/100 = 0.15, so we expect about 1000*0.15 = 150 darts will land there out of the 1000 thrown total. Each of those 150 darts in the red area represents a loss of $25. So we have a loss of 150*25 = 3750 dollars. We'll write this as -3750.

We also expect 0.65*1000 = 650 darts to have a gain of $5. That means 650*5 = 3250 is earned here. Furthermore, 0.20*1000 = 200 darts land in the green squares to get us $45 per attempt. So that's another 45*200 = 9000 dollars.

Overall, we would get -3750+3250+9000 = 8500 in profit.

Divide this over the 1000 dart throws and we end up with an expected value of 8500/1000 = 8.50

Keep in mind that this dart game or thought experiment is depending on theoretical probabilities. If you actually do this game, then your empirical probabilities will likely differ slightly from the theoretical values. However, the more trials you conduct, the closer your empirical probabilities should get to the theoretical ones.

Triangle N P O is shown. Line O N extends through point M to form exterior angle P N M. Angle N P O is 38 degrees. Angle P O N is 39 degrees. Exterior angle P N M is x degrees.
Which statement about the value of x is true?

x > 38
x < 39
x < 77
x > 103

Answers

Answer:

77

Step-by-step explanation:

i took the test

Answer:

c

Step-by-step explanation:

Solve the problem.
An initial investment of $930 is appreciated for 19 years in an account that earns
6% interest, compounded continuously. Find the amount of money in the
account at the end of the period.
$1275.35
$22,067.28
$12,424,545.93
$2907.89

Answers

Answer:

$2907.89

General Formulas and Concepts:

Symbols

e (Euler's number) ≈ 2.71828

Pre-Algebra

Order of Operations: BPEMDAS

Brackets Parenthesis Exponents Multiplication Division Addition Subtraction Left to Right

Algebra i

Compounded Continuously Formula: [tex]\displaystyle A = Pe^{rt}[/tex]

P is principle amountr is ratet is time

Step-by-step explanation:

Step 1: Define

Identify variables

P = 930

t = 19

r = 6% = 0.06

Step 2: Find Interest

Substitute in variables [Compounded Continuously Formula]:                    [tex]\displaystyle A = 930e^{19 \cdot 0.06}[/tex][Exponents] Multiply:                                                                                        [tex]\displaystyle A = 930e^{1.14}[/tex]Evaluate exponents:                                                                                         [tex]\displaystyle A = 930(3.12677)[/tex]Multiply:                                                                                                              [tex]\displaystyle A = 2907.89[/tex]

Please answer quickly, simple geometry

Answers

OPTION C IS THE CORRECT ANSWER.

What percent is represented by the shaded area?

Answers

Answer:91%

Step-by-step explanation:

help help help help help​

Answers

Step-by-step explanation:

press the picture to see the answer I swear it has a solution too

Luke's tyre pressure gauge shows a reading which is 8% higher than the actual pressure.
What is the actual pressure when Sue's gauge shows 39.42?

Answers

Answer:

so for this we do

39.42*1.08=42.5736

Hope This Helps!!!

Jude studied a total of 20.5 hours over a period of 5 days. On average, how many hours did Jude study each day

Answers

Question:Jude studied a total of 20.5 hours over a period of 5 days. On average, how many hours did Jude study each day?

Answer:4.1 hours

Explanation:20.5 ÷ 5 = 4.1Have a nice day! <3

Answer:

4.1 hours each day

Step-by-step explanation:

20.5 ÷ 5

= 4.1

Therefore 4.1 hours was spent on studying each day.

Name five whole numbers that can be expressed as the difference of two perfect squares. Show the math!

Answers

Answer : 25 & 16

Explanation :
25 - 16 = 9 or 5^2- 4^2 = 3
One pair is 25 and 16 and the other pair is 25 and 9.

I hope I helped u and answer your question :]

logo y = x means: log base b of y equals x.
True
False

Answers

If you meant to write [tex]\log_{b}(y) = x[/tex] then the statement is true.

As a more concrete example, something like [tex]\log_3(81) = 4[/tex] would be read out as "log base 3 of 81 is equal to 4".

Solve 7x + 6 <3(x - 2).
A. {xlx>-3}
B. {xlx<-3}
C. {xlx<-2}
D. {x1x<0}

Answers

Answer:

x<-3

Step-by-step explanation:

AC = 16, AB = x + 1, and BC = x + 7. What is the measure of the length of AB? HELP

Answers

Answer:

5

Step-by-step explanation:

AC=AB+BC

so 16=x+1+x+7

which simplifies to 16=2x+8

subtract eight from both sides to get 8=2x

then divide by 2 to get that x=4

AB=x+1, which substitutes into 4+1=5

Please Help!!!! Please!!!!!!!!

Answers

9514 1404 393

Answer:

  see attached

Step-by-step explanation:

I found it easiest to draw arcs of radii AB and AC centered at A, and arcs of radii DB and DC centered at D. The intersection points of those arcs mark the points B' and C'. These are points B and C reflected across line AD.

Joining A, B', C', and D will make a figure symmetrical about AD.

X 8. From the given Venn-diagram, list the elements of the following sets, a) U b) XU Y and XUY c) X Y and X d) X - Y and X-Y e) Y - X and Y - X 11 15

Answers

Answer:

where is the Venn diagram plz insert Venn diagram then I should solve

PLEASE HELP

What are the missing reasons in the proof?
A. Substitution; converse of diagonals theorem
B. Sum of interior angles; converse of diagonals theorem
C. Sum of interior angles; converse of opposite angles theorem
D. Substitution; converse of opposite angles theorem

Answers

Are these converses true? The answer is yes. Each of these converses can be a way to show that a quadrilateral is a parallelogram. However, the Consecutive Angles Converse can be a bit tricky, considering you would have to show that each angle is supplementary to its neighbor

(∠A

and

∠B, ∠B

and

∠C, ∠C

and

∠D

, and

∠A

and

∠D)

. We will not use this converse.

Consider the series 1/4+1/6+1/9+2/27+4/81+....


Does the series converge or diverge?


Select answers from the drop-down menus to correctly complete the statements.

Answers

Answer:

The series converges as any two consecutive elements of the sum are monotonically decreasing.

Step-by-step explanation:

The series converges since the consecutive element is monotonically decreasing. That is:

[tex]\forall\,i\in \mathbb{N}\,a_{i} > a_{i+1}[/tex] (1)

Where:

[tex]a_{i}[/tex] - The i-th component of the sum.

[tex]a_{i+1}[/tex] - The (i+1)-th component of the sum.

Help for brainliest
Find h.
9 ft
h = [?] ft

Answers

Answer:

[tex]\sqrt{77}[/tex] ft

Step-by-step explanation:

Hi there!

Notice how we can create a right triangle in the cone, where the base is 2 ft (4/2=2), the height is h and the hypotenuse is 9 ft.

Using these values, we can solve for h with the Pythagorean theorem: [tex]a^2+b^2=c^2[/tex] where a and b are the two legs of a right triangle and c is the hypotenuse.

Plug in 2 and h as a and b and 9 as c:

[tex]2^2+h^2=9^2\\4+h^2=81\\h^2=81-4\\h^2=77\\h=\sqrt{77}[/tex]

I hope this helps!

Of the marbles in a​ bag, 4 are yellow ​, 4 are white ​, and 5 are green . Sandra will randomly choose one marble from the bag. Fill in the blanks in parts a and b. The probability that Sandra will choose a marble from the bag is

Answers

Answer:

Probability[Random choose marble is green] = 5 / 13

Step-by-step explanation:

Given:

Number of yellow marble in bag = 4

Number of white marble in bag = 4

Number of green marble in bag = 5

Find:

Probability[Random choose marble is green]

Computation:

Probability of an event = Number of favourable outcome / Number of total outcome

Probability[Random choose marble is green] = 5 / [4 + 4 + 5]

Probability[Random choose marble is green] = 5 / 13

35/48*100 fraction into percentage.​

Answers

Answer:

729167

Step-by-step explanation:

Now we can see that our fraction is 72.916666666667/100, which means that 35/48 as a percentage is 72.9167%.

Select all the correct answers.
Which expressions are equivalent to the given expression?

Answers

Answer:

6√7=15.874507866387543=√252

Step-by-step explanation:

hope this is helpful

The expression that are equivalent to √252 are [tex]252^{\frac{1}{2} }[/tex] and 6√7.

How to find equivalent expression?

Applying the surd rule,

√a = [tex]a^{\frac{1}{2} }[/tex]

Hence,

[tex]\sqrt{252}=252^{\frac{1}{2} }[/tex]

using surd rule, we can also decompose √252

Therefore,

√252 = √36 × 7 = 6√7

Hence, the equivalent expression of √252 are as follows:

[tex]252^{\frac{1}{2} }[/tex]6√7

learn more on expression here: https://brainly.com/question/27768447

#SPJ2

Convert the following decimal to fraction 3.04

Answers

Answer:

3 1/25

Step-by-step explanation:

3.04

Do the decimal part first

.04 = 4/100

Reduce - cancel out 4

1/25

Put back the whole number

3 1/25

Answer:

3 [tex]\frac{1}{25}[/tex]

Step-by-step explanation:

I agree with the previous answer :

You need to do it part by part.

0.04 is equal to [tex]\frac{4}{100}[/tex]

[tex]\frac{4}{100}[/tex] simplified (we divide by 4 with both the denomantor and the numerator.) =

[tex]\frac{1}{25}[/tex]

And we still have those 3 wholes that we had in the beginning.

Now we add.

3 + [tex]\frac{1}{25}[/tex] = 3 [tex]\frac{1}{25}[/tex]

Which table of ordered pairs represents a proportional relationship?
х
-2
4
-6
y
4
16
36
х
-4
-6
-8
y
-8
-12
-18
o
х
-3
-5
-7
y
5
3
1
O
х
-3
-6
-9
y
12
24
36

Answers

Answer:

x=-3×-5×-7

-3×-5=15

15×-7=-105

=-105

Other Questions
Three forces are pulling on the same object such that the system is in equilibrium. Their magnitudes are F1 = 2.83 N.F= 3.35 N. and F3 = 3.64 N, and they make angles of 0, = 45.0, 02 = -63.43 and 03 =164.05 with respect to the x-axis, respectively.Required:a. What is the x-component of the force vector F1?b. What is the y-component of the force vector F1? please help me please help me please help me please help me please help me please help me please Look the photoplz help me asapi will give brainliest What impact does sitting have on the brain? What are your classes like? What are your teachers like? Select three classes and write a sentence describing each of them (for a total of three sentences). Then select two teachers and write a sentence describing each of them (for a total of two sentences). You should have a total of five complete sentences in Spanish. I have been stuck on this all day, please explain how to do this Cmo sobreviviran las personas, sin gas, agua, luz, telfono, ni internet? 3. A student gets a grant of $10 000 a year. Assuming her grant is increased by 7% each year, what will her grant be in four years time? Jaheem has a goal running a total of 125 miles this month. Each day that he ran, he ran 7 miles. Which expression could Jaheem use to determine how many miles he has left to run after running for d days?x a.125 7dx b.7d + 125x c. fraction numerator 125 over denominator 7 d end fractionx d.7d The figure below is made of 2 rectangular prisms.What is the volume of this figure? Choose the Athat seems to be congruent to the given one.R.FDBAEGFAOEGDo CGDBGC who are the frontliners?? The reason that the mail bed appears pink is the presence of a larger number of melanocytes in the underlying dermis The problem has been started for you.What is the quotient?8686.686.ModifyingAbove 6 with bar87 A chewing gum consists of 30% corn syrup and 40% sugar. The rest was made up by gum Arabic. Find the weight of the Gum Arabic in a 25gm piece of the chewing gum. Max points pls just bloody answer it If a +b+c=9 and a square +b square +square =1 then ab+bc+ca Elena receives $131 per year in simple interest from three investments totaling $3000. Part is invested at 3%, part at 4% and part at 5%. There is $1000 more invested at 5% than at 4%. Find the amount invested at each rate.The amount invested at 3% is $___ the amount invested at 4% is $___ , and the amount invested at 5% is $___ Monetary policy refers to Question 23 options: actions taken by banks and other financial institutions regarding their approaches to lending, account management, etc. changes in the money supply to achieve particular economic goals. changes in government expenditures and taxation to achieve particular economic goals. the change in private expenditures that occurs as a consequence of changes in the money supply. Congruent angle pairs : find value of x and the value of y Which Stage has the most specialized cells.